Вы находитесь на странице: 1из 19

Republic of the Philippines

Department of Education
Region X
Division of Cagayan de Oro City
GUSA REGIONAL SCIENCE HIGH SCHOOL - X
Gusa, Cagayan de Oro City

BASIC CALCULUS RECAP


(PROBLEM AND SOLUTION)
TOPIC: LIMITS AND DISCONTINUITY

Submitted by:
Jodi Elle P. Cagampang
11 - Resilience

Submitted to:
Sir Ferdinand Corpuz
Basic Calculus Teacher
LIMITS OF FUNCTIONS

1. Find the limit

Solution to Example 1:
x<1
x-1<0
If x - 1 < 0 then
| x - 1 | = - (x - 1)
Substitute | x - 1 | by - (x - 1), factor the numerator to write the limit as follows

Simplify to obtain

2. Find the limit

Solution to Example 2:
Although the limit in question is the ratio of two polynomials, x = 5 makes both the
numerator and denominator equal to zero. We need to factor both numerator and
denominator as shown below.

Simplify to obtain

3. Calculate the limit

Solution to Example 3:
We need to look at the limit from the left of 2 and the limit from the right of 2. As x
approaches 2 from the left x - 2 < 0; hence |x - 2| = - (x - 2).Substitute to obtain the limit from
the left of 2 as follows
=-8

As x approaches 2 from the right x - 2 > 0 hence


|x - 2| = x - 2
Substitute to obtain the limit from the right of 2 as follows

= 8, The limit from the right of 2 and the limit from the left of 2

are not equal therefore the given limit DOES NOT EXIST.
4. Calculate the limit

Solution to Example 4:
As x approaches -1, cube root x + 1 approaches 0 and ln (x+1) approaches - infinity hence an
indeterminate form 0 . infinity

Let us rewrite the limit so that it is of the infinity/infinity indeterminate form.

We now use L'hopital's Rule and find the limit.

5. Find the limit

Solution to Example 5:
As x gets larger x + 1 gets larger and e^(1/(x+1)-1) approaches 0 hence an indeterminate
form infinity.0

Let us rewrite the limit so that it is of the 0/0 indeterminate form.

Apply the l'hopital's theorem to find the limit.

=-1
6. Find the limit

Solution to Example 6:

As x approaches 9, both numerator and denominator approach 0. Multiply both


numerator and denominator by the conjugate of the numerator.

Expand and simplify.

and now find the limit.

=1/6
7. Find the limit

Solution to Example 7:
The range of the cosine function is.
-1 <= cos x <= 1
Divide all terms of the above inequality by x, for x positive.
-1 / x <= cos x / x <= 1 / x
Now as x takes larger values without bound (+infinity) both -1 / x and 1 / x approaches 0.
Hence by the squeezing theorem the above limit is given by

8. Find the limit

Solution to Example 8:
As t approaches 0, both the numerator and denominator approach 0 and we have the 0 / 0
indeterminate form. Hence the l'hopital theorem is used to calculate the above limit as
follows

Find the limit

9.
Solution to Example 9:
We first factor out 16 x 2 under the square root of the denominator and take out of the
square root and rewrite the limit as

Since x approaches larger positive values (infinity) | x | = x.


Simplify and find the limt.

=3/4
10. Find the limit

Solution to Example 10:


As x approaches 2 from the left then x - 2 approaches 0 from the left or x - 2 < 0. The
numerator approaches 5 and the denominator approaches 0 from the left hence the limit is
given by

CONTINUITY OF A FUNCTION

1.

2.

3.

4. Determine if the function is continuous at x=-1

5.

6.

7.

8.

9.

10.
Answers:
1.

,is not satisfied and function f is NOT continuous at x=1 .


2.

does not exist, Thus, function f is NOT continuous at x=-2 .


3.

all three conditions are satisfied, and f is continuous at x=0.


4. Function h is not defined at x=-1 since it leads to division by zero. Thus,h(-1)does not
exist, condition is violated, and function h is NOT continuous at x = -1 .
5. f is NOT continuous at x=-3 .
6. Since (x-1)(x+4) = 0 for x=1 and x=-4 , function f is continuous for all values
of x EXCEPT x=1 and x=-4

7. , function g is continuous for all values


of x .
8. function f is continuous for and .
9. function f is continuous for x < -2 and x > 1 .

10. Summarizing, the quotient of these continuous functions, , is


continuous for and , but NOT for x = 5 and x = -5 .

EXPONENTIAL

1. Differentiate the function y = e sin x

2. Differentiate the function y = e–3xsin4x

3. Differentiate y = x3 + 3x
4. Differentiate y = 52x+1

5. y=e−x3

y′=(e−x3)′=e−x3⋅(−x3)′=e−x3⋅(−3x2)=−3x2e−x3.

6. y=43x2

y′=(43x2)′=43x2ln4⋅(3x2)′=43x2ln4⋅6x=6x43x2ln4.

7. y=2−x2

y′=(2−x2)′=2−x2ln2⋅(−x2)′=2−x2ln2⋅(−2x)=−x21−x2ln2.

8. y=4x⋅32x

y′(x)=(4x⋅32x)′=(36x)′=36xln36.

9. y=x22x.

y′=(x22x)′=(x2)′⋅2x+x2⋅(2x)′=2x⋅2x+x2⋅2xln2=x2x(2+xln2).

10. y=ex(x2−2x+2)

=ex(x2−2x+2)+ex(2x−2)=ex(x2−2x+2+2x−2)=exx2

LIMIT THEOREMS

1.

2.
3.

43 + 4 = 64 + 4 = 68. This follows from Theorem 1 and Theorem 2.

4.
42 + 1 = 16 + 1 = 17. This follows from Theorem 1, Theorem 2, and Theorem 4.

5.

5· 42 = 5· 16 = 80. This follows from Theorem 5 and Theorem 2.

6.
15
5 = 3.
The limits of the numerator and denominator follow from Theorems 1, 2,
and 4. The limit of the fraction follows from Theorem 3.

7. Evaluate

3(−1)2 −5(−1) + 1 = 3 + 5 + 1 = 9.

8.
On replacing h with 0, the limit is 4x3.

9.

10.

PROVING LIMITS

1. Use the definition of the limit to prove the following


limit.limx→0x2=0limx→0⁡x2=0

In this case both LL and aa are zero. So, let ε>0ε>0 be any number. Don’t
worry about what the number is, εε is just some arbitrary number. Now
according to the definition of the limit, if this limit is to be true we will need to
find some other number δ>0δ>0 so that the following will be true.

∣∣x2−0∣∣<εwhenever0<|x−0|<δ|x2−0|<εwhenever0<|x−0|<δ
Or upon simplifying things we need,

∣∣x2∣∣<εwhenever0<|x|<δ|x2|<εwhenever0<|x|<δ

Often the way to go through these is to start with the left inequality and do a
little simplification and see if that suggests a choice for δδ. We’ll start by
bringing the exponent out of the absolute value bars and then taking the
square root of both sides.

|x|2<ε⇒|x|<√ ε |x|2<ε⇒|x|<ε

Now, the results of this simplification looks an awful lot


like 0<|x|<δ0<|x|<δ with the exception of the “0<0<” part. Missing that however
isn’t a problem, it is just telling us that we can’t take x=0x=0. So, it looks like if
we choose δ=√ ε δ=ε we should get what we want.

We’ll next need to verify that our choice of δδ will give us what we want, i.e.,

∣∣x2∣∣<εwhenever0<|x|<√ ε |x2|<εwhenever0<|x|<ε

Verification is in fact pretty much the same work that we did to get our guess.
First, let’s again let ε>0ε>0 be any number and then choose δ=√ ε δ=ε. Now,
assume that 0<|x|<√ ε 0<|x|<ε. We need to show that by choosing xx to satisfy
this we will get,

∣∣x2∣∣<ε|x2|<ε

To start the verification process, we’ll start with ∣∣x2∣∣|x2| and then first strip out
the exponent from the absolute values. Once this is done we’ll use our
assumption on xx, namely that |x|<√ ε |x|<ε. Doing all this gives,

∣∣x2∣∣=|x|2strip exponent out of absolute value bars < (√ ε )2use the assumption
that |x|<√ ε =εsimplify|x2|=|x|2strip exponent out of absolute value
bars < (ε)2use the assumption that |x|<ε=εsimplify

Or, upon taking the middle terms out, if we assume that 0<|x|<√ ε 0<|x|<ε then
we will get,

∣∣x2∣∣<ε|x2|<ε

and this is exactly what we needed to show.

So, just what have we done? We’ve shown that if we choose ε>0ε>0 then we
can find a δ>0δ>0 so that we have,

∣∣x2−0∣∣<εwhenever0<|x−0|<√ ε |x2−0|<εwhenever0<|x−0|<ε
and according to our definition this means that,

limx→0x2=0

2. Use the definition of the limit to prove the following


limit.limx→25x−4=6limx→2⁡5x−4=6

We’ll start this one out the same way that we did the first one. We won’t be
putting in quite the same amount of explanation however.

Let’s start off by letting ε>0ε>0 be any number then we need to find a
number δ>0δ>0 so that the following will be true.

|(5x−4)−6|<εwhenever0<|x−2|<δ|(5x−4)−6|<εwhenever0<|x−2|<δ

We’ll start by simplifying the left inequality in an attempt to get a guess for δδ.
Doing this gives,

|(5x−4)−6|=|5x−10|=5|x−2|<ε⇒|x−2|<ε5|(5x−4)−6|=|5x−10|=5|x−2|<ε⇒|x−2|<ε5

So, as with the first example it looks like if we do enough simplification on the
left inequality we get something that looks an awful lot like the right inequality
and this leads us to choose δ=ε5δ=ε5.

Let’s now verify this guess. So, again let ε>0ε>0 be any number and then
choose δ=ε5δ=ε5. Next, assume that 0<|x−2|<δ=ε50<|x−2|<δ=ε5 and we get
the following,

|(5x−4)−6|=|5x−10|simplify things a little=5|x−2|more simplification....<5(ε5)use


the assumption δ=ε5=εand some more simplification|(5x−4)−6|=|5x−10|simplify
things a little=5|x−2|more simplification....<5(ε5)use the
assumption δ=ε5=εand some more simplification
|(5x−4)−6|<εwhenever0<|x−2|<ε5|(5x−4)−6|<εwhenever0<|x−2|<ε5

and so by our definition we have,

limx→25x−4=6

3. Use the definition of the limit to prove the following


limit.limx→4x2+x−11=9limx→4⁡x2+x−11=9

So, let’s get started. Let ε>0ε>0 be any number then we need to find a
number δ>0δ>0 so that the following will be true.

∣∣(x2+x−11)−9∣∣<εwhenever0<|x−4|<δ|(x2+x−11)−9|<εwhenever0<|x−4|<δ

We’ll start the guess process in the same manner as the previous two
examples.
∣∣(x2+x−11)−9∣∣=∣∣x2+x−20∣∣=|(x+5)(x−4)|=|x+5||x−4|<ε|(x2+x−11)−9|=|x2+x−20
|=|(x+5)(x−4)|=|x+5||x−4|<ε

Okay, we’ve managed to show that ∣∣(x2+x−11)−9∣∣<ε|(x2+x−11)−9|<ε is


equivalent to |x+5||x−4|<ε|x+5||x−4|<ε. However, unlike the previous two
examples, we’ve got an extra term in here that doesn’t show up in the right
inequality above. If we have any hope of proceeding here we’re going to need
to find some way to deal with the |x+5||x+5|.

To do this let’s just note that if, by some chance, we can show
that |x+5|<K|x+5|<K for some number KK then, we’ll have the following,

|x+5||x−4|<K|x−4||x+5||x−4|<K|x−4|

If we now assume that what we really want to show


is K|x−4|<εK|x−4|<ε instead of |x+5||x−4|<ε|x+5||x−4|<ε we get the following,

|x−4|<εK|x−4|<εK

This is starting to seem familiar isn’t it?

All this work however, is based on the assumption that we can show
that |x+5|<K|x+5|<K for some KK. Without this assumption we can’t do
anything so let’s see if we can do this.

Let’s first remember that we are working on a limit here and let’s also
remember that limits are only really concerned with what is happening around
the point in question, x=4x=4 in this case. So, it is safe to assume that
whatever xx is, it must be close to x=4x=4. This means we can safely assume
that whatever xx is, it is within a distance of, say one of x=4x=4. Or in terms of
an inequality, we can assume that,

|x−4|<1|x−4|<1

Why choose 1 here? There is no reason other than it’s a nice number to work
with. We could just have easily chosen 2, or 5, or 1313. The only difference our
choice will make is on the actual value of KK that we end up with. You might
want to go through this process with another choice of KK and see if you can
do it.

So, let’s start with |x−4|<1|x−4|<1 and get rid of the absolute value bars and
this solve the resulting inequality for xx as follows,

−1<x−4<1⇒3<x<5−1<x−4<1⇒3<x<5

If we now add 5 to all parts of this inequality we get,

8<x+5<108<x+5<10
Now, since x+5>8>0x+5>8>0 (the positive part is important here) we can say
that, provided |x−4|<1|x−4|<1 we know that x+5=|x+5|x+5=|x+5|. Or, if take the
double inequality above we have,

8<x+5=|x+5|<10⇒|x+5|<10⇒K=108<x+5=|x+5|<10⇒|x+5|<10⇒K=10

So, provided |x−4|<1|x−4|<1 we can see that |x+5|<10|x+5|<10 which in turn


gives us,

|x−4|<εK=ε10|x−4|<εK=ε10

So, to this point we make two assumptions about |x−4||x−4| We’ve assumed
that,

|x−4|<ε10AND|x−4|<1|x−4|<ε10AND|x−4|<1

It may not seem like it, but we’re now ready to choose a δδ. In the previous
examples we had only a single assumption and we used that to give us δδ. In
this case we’ve got two and they BOTH need to be true. So, we’ll let δδ be the
smaller of the two assumptions, 1 and ε10ε10. Mathematically, this is written
as,

δ=min{1,ε10}δ=min{1,ε10}

By doing this we can guarantee that,

δ≤ε10ANDδ≤1δ≤ε10ANDδ≤1

Now that we’ve made our choice for δδ we need to verify it. So, ε>0ε>0 be any
number and then chooseδ=min{1,ε10}δ=min{1,ε10}. Assume
that 0<|x−4|<δ=min{1,ε10}0<|x−4|<δ=min{1,ε10}. First, we get that,

0<|x−4|<δ≤ε10⇒|x−4|<ε100<|x−4|<δ≤ε10⇒|x−4|<ε10

We also get,

0<|x−4|<δ≤1⇒|x−4|<1⇒|x+5|<100<|x−4|<δ≤1⇒|x−4|<1⇒|x+5|<10

Finally, all we need to do is,

∣∣(x2+x−11)−9∣∣=∣∣x2+x−20∣∣simplify things a little=|x+5||x−4|factor<10|x−4|use


the assumption that |x+5|<10<10(ε10)use the assumption that |x−4|<ε10<εa little
final simplification|(x2+x−11)−9|=|x2+x−20|simplify things a
little=|x+5||x−4|factor<10|x−4|use the assumption that |x+5|<10<10(ε10)use
the assumption that |x−4|<ε10<εa little final simplification

We’ve now managed to show that,

∣∣(x2+x−11)−9∣∣<εwhenever0<|x−4|<min{1,ε10}|(x2+x−11)−9|<εwhenever0<|x
−4|<min{1,ε10}
and so by our definition we have,

limx→4x2+x−11=9

4. Use the definition of the limit to prove the following


limit.limx→0+√ x =0limx→0+⁡x=0

Let ε>0ε>0 be any number then we need to find a number δ>0δ>0 so that the
following will be true.

∣∣√ x −0∣∣<εwhenever0<x−0<δ|x−0|<εwhenever0<x−0<δ

Or upon a little simplification we need to show,

√ x <εwhenever0<x<δx<εwhenever0<x<δ

As with the previous problems let’s start with the left-hand inequality and see if
we can’t use that to get a guess for δδ. The only simplification that we really
need to do here is to square both sides.

√ x <ε⇒x<ε2x<ε⇒x<ε2

So, it looks like we can choose δ=ε2δ=ε2.

Let’s verify this. Let ε>0ε>0 be any number and chose δ=ε2δ=ε2. Next assume
that 0<x<ε20<x<ε2. This gives,

∣∣√ x −0∣∣=√ x some quick simplification<√ ε2 use the assumption that x<ε2<εone
final simplification|x−0|=xsome quick simplification<ε2use the assumption
that x<ε2<εone final simplification

We’ve now shown that,

∣∣√ x −0∣∣<εwhenever0<x−0<ε2|x−0|<εwhenever0<x−0<ε2

and so by the definition of the right-hand limit we have,

limx→0+√ x =0

5. Use the definition of the limit to prove the following


limit.limx→01x2=∞limx→0⁡1x2=∞

These work in pretty much the same manner as the previous set of examples
do. The main difference is that we’re working with an MM now instead of
an εε. So, let’s get going.
Let M>0M>0 be any number and we’ll need to choose a δ>0δ>0 so that,

1x2>Mwhenever0<|x−0|=|x|<δ1x2>Mwhenever0<|x−0|=|x|<δ

As with the all the previous problems we’ll start with the left inequality and try
to get something in the end that looks like the right inequality. To do this we’ll
basically solve the left inequality for xx and we’ll need to recall
that √ x2 =|x|x2=|x|. So, here’s that work.

1x2>M⇒x2<1M⇒|x|<1√ M 1x2>M⇒x2<1M⇒|x|<1M

So, it looks like we can choose δ=1√ M δ=1M. All we need to do now is verify
this guess.

Let M>0M>0 be any number, choose δ=1√ M δ=1M and assume


that 0<|x|<1√ M 0<|x|<1M.

In the previous examples we tried to show that our assumptions satisfied the
left inequality by working with it directly. However, in this, the function and our
assumption on xx that we’ve got actually will make this easier to start with the
assumption on xx and show that we can get the left inequality out of that. Note
that this is being done this way mostly because of the function that we’re
working with and not because of the type of limit that we’ve got.

Doing this work gives,

|x|<1√ M |x|2<1Msquare both sidesx2<1Macknowledge that |x|2=x21x2>Msolve


for x2|x|<1M|x|2<1Msquare both sidesx2<1Macknowledge
that |x|2=x21x2>Msolve for x2

So, we’ve managed to show that,

1x2>Mwhenever0<|x−0|<1√ M 1x2>Mwhenever0<|x−0|<1M

and so by the definition of the limit we have,

limx→01x2=∞

6. Use the definition of the limit to prove the following


limit.limx→−∞1x=0limx→−∞⁡1x=0

Let ε>0ε>0 be any number and we’ll need to choose a N<0N<0 so that,

∣∣∣1x−0∣∣∣=1|x|<εwheneverx<N|1x−0|=1|x|<εwheneverx<N

Getting our guess for NN isn’t too bad here.

1|x|<ε⇒|x|>1ε1|x|<ε⇒|x|>1ε
Since we’re heading out towards negative infinity it looks like we can
choose N=−1εN=−1ε. Note that we need the “-” to make sure that NN is
negative (recall that ε>0ε>0).

Let’s verify that our guess will work. Let ε>0ε>0 and choose N=−1εN=−1ε and
assume that x<−1εx<−1ε. As with the previous example the function that we’re
working with here suggests that it will be easier to start with this assumption
and show that we can get the left inequality out of that.

x<−1ε|x|>∣∣∣−1ε∣∣∣take the absolute value|x|>1εdo a little simplification1|x|<εsolve


for |x|∣∣∣1x−0∣∣∣<εrewrite things a littlex<−1ε|x|>|−1ε|take the absolute
value|x|>1εdo a little simplification1|x|<εsolve for |x||1x−0|<εrewrite things a
little

Note that when we took the absolute value of both sides we changed both
sides from negative numbers to positive numbers and so also had to change
the direction of the inequality.

So, we’ve shown that,

∣∣∣1x−0∣∣∣=1|x|<εwheneverx<−1ε|1x−0|=1|x|<εwheneverx<−1ε

and so by the definition of the limit we have,

limx→−∞1x=0

7. Prove that . Begin by letting be given. Find so that


if , then , i.e., , i.e., . But this trivial
inequality is always true, no matter what value is chosen for . For

example, will work. Thus, if , then it follows that .


This completes the proof.

8. Prove that . Begin by letting be given. Find (which


depends on ) so that if , then . Begin
with and ``solve for" |x-10| . Then,

iff

iff

iff

iff
iff .

Now choose . Thus, if , it follows that . This


completes the proof.

9. Prove that . Begin by letting be given. Find (which


depends on ) so that if , then . Begin
with and ``solve for" . Then,

iff

iff

iff

iff

iff

iff .

Now choose . Thus, if , it follows that


. This completes the proof.

10. Prove that . Begin by letting be given. Find (which


depends on ) so that if , then . Begin
with and ``solve for" |x-1| . Then,

iff

iff

iff

iff .

We will now ``replace" the term |x+1| with an appropriate constant and keep the term
|x-1| , since this is the term we wish to ``solve for". To do this, we will arbitrarily
assume that (This is a valid assumption to make since, in general, once we find
a that works, all smaller values of also work.) . Then implies that -1
< x-1 < 1 and 0 < x < 2 so that 1 < |x+1| < 3 (Make sure that you understand this step
before proceeding.). It follows that (Always make this ``replacement" between your
last expression on the left and . This guarantees the logic of the proof.)

iff

iff .

Now choose (This guarantees that both assumptions made about in


the course of this proof are taken into account simultaneously.). Thus,
if , it follows that . This completes the proof.

Infinite Limits

1. limx→0+1x = limx→0+1x=∞

2. limx→0−1x = limx→0−1x=−∞

3. limx→01x= limx→01x doesn't exist

4. limx→0+6x2 = limx→0+6x2=∞

5. limx→0−6x2= limx→0−6x2=∞

6. limx→06x2= limx→06x2=∞

7. limx→0+ln(x) = limx→0+ln(x)=−∞

8. limx→π2+tan(x) = limx→π2+tan(x)=−∞

9. limx→π2−tan(x) = limx→π2−tan(x)=∞

10. limx→4−3(4−x)3 = limx→4−3(4−x)3=∞

One-Sided Limits

1. limx→−4−f(x)≈ limx→−4−f(x)limx→−4−f(x) does not exist.


2. limx→−4+f(x)≈ limx→−4+f(x)≈−2
3. limx→1−f(x)≈limx→1−f(x)≈ limx→1−f(x)≈−3
4. limx→1−f(x) based: Let f(x)={2−x(x−2)20<x<11<x<2,
f(x)={2−x0<x<1(x−2)21<x<2
As xx approaches 1 from the left, we see that f(x)f(x) approaches 1. Therefore limx→1−f(x)=1.
5. limx→1+f(x)

As xx approaches 1 from the right, we see that again f(x)f(x) approaches 1.


Therefore limx→1+f(x)=1limx→1+f(x)=1.
6. limx→1f(x)

As xx approaches 1 from the right, we see that again f(x)f(x) approaches 1.


Therefore limx→1+f(x)=1limx→1+f(x)=1.
7. f(1)

f(1)f(1) is not defined. Note that 1 is not in the domain of ff as defined by the problem, which is
indicated on the graph by an open circle when x=1x=1.
8. limx→0+f(x)

As xx goes to 0 from the right, f(x)f(x) approaches 2. So limx→0+f(x)=2limx→0+f(x)=2.


9. limx→2−f(x)

As xx goes to 2 from the left, f(x)f(x) approaches 0. So limx→2−f(x)=0limx→2−f(x)=0.


10. f(2)
f(2)f(2) is not defined as 2 is not in the domain of ff.

Two-Sided Limits

Answers:

9) 3/5 10) 3/5

Вам также может понравиться